You are on page 1of 4

BÀI TẬP SỐ HỌC NGÀY 20+21/9

Bài 1. Tìm các số nguyên dương x, y sao cho x 2 + 1 ⋮ y 2 - 5


Lời giải.
Gọi p là một ước nguyên tố của y 2 - 5 . Ta có x 2 ≡ - 1(mod p) thì khi đó
p-1
-1
= (-1) 2 =1
p
Nên p = 4k + 1 . Dẫn tới y 2 - 5 ≡ 1(mod 4) nhưng y 2 - 5 ≡ 3, 0(mod 4) nên ta có điều vô
lý. Vậy, không tồn tại x, y thỏa mãn
2
Bài 2. Tìm tất cả các số nguyên tố p sao cho 5 p + 1 ≡ 0 mod p 2 .
Lời giải.
2
Ta có 5 2p ≡ 1 mod p 2 . Ngoài ra theo định lý Fermat bé thì

5 p-1 ≡ 1(mod p)
Ta gọi h = ord p 5 thì khi đó h|2p 2 và h|p - 1 . Nhận xét rằng (p - 1, p) = 1 do p là số nguyên
tố. Khi đó h| 2p 2 , p - 1 = 2 . Ta xét 2 trường hợp
Nếu h = 1 thì 5 ≡ 1(mod p) hay p = 2 nhưng thử lại thì không đúng
Nếu h = 2 thì 15 ⋮ p nên p = { 3, 5 } . Thử trực tiếp ta có p = 3 thỏa.
Bài 3. Chứng minh với mọi số nguyên tố p thì tồn tại vô số số nguyên tố n sao cho 2 n - n ⋮ p
Lời giải.
Nếu p = 2 thì ta chỉ cần chọn sao cho n chẵn là đủ
Nếu p lẻ thì ta có 2 n ≡ n(mod p), ngoài ra 2 p-1 ≡ 1(mod p) dẫn tới 2 p-1 - 1 ⋮ p . Tức là
bây
giờ ta phải chọn n sao cho
n ≡ 1(mod p)
Và ngoài ra 2 m(p-1) ≡ 1(mod p). Ta chọn m sao cho m ≡ -1(mod p) thì khi đó ta sẽ cho
n = m(p - 1) sẽ được n ≡ 1(mod p). Và bài toán hoàn tất do tập hợp số nguyên tố là vô
hạn nên ta có thể chọn vô số số m như vậy.
Bài 4. Cho p là số nguyên tố khác 2 và a, b là hai số tự nhiên lẻ sao cho a + b chia hết cho p
và a - b chia hết cho p - 1 . Chứng minh a b + b a chia hết cho 2p
Lời giải.
Do a - b chia hết cho p - 1 nên a - b = k(p - 1) . a, b lúc này đây có thể cùng chia hết cho p
được nên ta không thể dùng định lý Fermat bé để đánh giá được. Thế thì ta có thể nghĩ tới 1
ý tưởng sau đây
Ý tưởng 1 : Xét trường hợp
Ta sẽ xét nếu a, b đều chia hết cho p thì điều phải chứng minh hiển nhiên đúng.
Nếu a, b không có số nào chia hết cho p thì khi đó theo định lý Fermat nhỏ ta có
a p-1 ≡ b p-1 ≡ 1(mod p)
Ngoài ra a ≡ -b(mod p) nên a b ≡ - b b nên a b + b a ≡ b a - b b = b b b a-b - 1 .
k
Khi đó b a-b = b k(p-1) = b p-1 ≡ 1(mod p) nên ta có điều phải chứng minh.
9p - 1
Bài 5. Cho p là một số nguyên tố lẻ. Đặt m = . Chứng minh rằng m là một hợp số lẻ,
8
không chia hết cho 3 và 3 m-1 ≡ 1(mod m )
Lời giải
9p - 1
Ta có m = = 9 p-1 + 9 p-2 + ... + 9 + 1 , có p số lẻ nên tổng của chúng là một số lẻ.
p-1
Ngoài ra

3p - 1 3p + 1 3p - 1 3p + 1
m= = .
8 4 2
p p
3 -1 3 +1
Mà cả hai số , > 1 nên m là hợp số.
4 2
p
9p - 9
Theo định lý Fermat nhỏ thì ta có 9 ≡ 9(mod p). Ngoài ra m - 1 = . Ta có
8
9 p - 1 ⋮ m . Dẫn tới 3 2p - 1 ⋮ m . Mà ta có 3 m-1 - 1 ⋮ 3 2p - 1 , vì m - 1 ⋮ p và m - 1 ⋮ 2 . Suy
ra ta được đẳng thức đồng dư sau
3 m-1 - 1 ≡ 3 2p - 1 ≡ 0(mod m)
Vậy ta có điều phải chứng minh
4p - 1
Bài 6. Cho p > 3 là số nguyên tố và m = . Chứng minh 2 m-1 ≡ 1(mod m)
3

Lời giải
2p + 1 p
Ta làm y hệt bài 5, trước hết thì ta có m = 2 - 1 nên m là một hợp số và m cũng
3
4p - 4
là số lẻ. Khi đó ta có m - 1 = và 4 p - 4 ⋮ p nhưng do m lẻ nên m - 1 ⋮ 2 dẫn tới
3
m - 1 ⋮ 2p . Mặt khác 2 2p - 1 ⋮ m nên
2 m-1 - 1 ⋮ 2 2p - 1 ⋮ m
Cuối cùng có 2 m-1 ≡ 1(mod m)
ap - 1
Ta có thể tổng quát lên m = với p là số nguyên tố và a = t 2 . Khi đó t m-1 ≡ 1(mod m)
a-1
Bài 7. Cho số nguyên tố p và số nguyên a > 1. Chứng minh rằng nếu q là một ước nguyên
ap - 1
tố của thì q = p hoặc q ≡ 1(mod p).
a-1
Lời giải.
Ta rõ ràng có a p ≡ 1(mod q) nên (a, q) = 1 . Suy ra a q-1 ≡ 1(mod q). Gọi h = ord q (a) thì
h|p và h|q - 1 nên h|(p, q - 1) . Nếu h = 1 thì
ap - 1
= a p-1 + ... + 1 ≡ p(mod q)
a-1
Nên suy ra p = q. Nếu h = p thì h|q - 1 tức là q - 1 ≡ 0(mod p) hay q = kp + 1 và ta có
điều phải chứng minh
Bài 8. Cho các số nguyên tố p, q, r trong đó p lẻ và thỏa p|q r + 1 thì khi đó 2r|p - 1 hoặc
p|q 2 - 1
Lời giải
Ta có q r ≡ - 1(mod p) và do đó (p, q) = 1 nên q p-1 ≡ 1(mod p). Ngoài ra
q 2r ≡ 1(mod p). Nếu ta gọi h = ordp q thì h|p - 1 và h|2r. Vậy h|(p - 1, 2r) . Chú ý do r là số
nguyên tố nên h ∈ { 1, 2, r, 2r } .
Nếu h = 1 thì q ≡ 1(mod p) tức q - 1 ⋮ p ( đúng)
Nếu h = 2 thì q 2 - 1 ⋮ p ( đúng)
Nếu h = r thì q h ≡ 1(mod p) khi đó p - 1 ⋮ r và ngoài ra do p lẻ nên p - 1 ⋮ 2 dẫn tới
p - 1 ⋮ 2r
Nếu h = 2r thì dẫn tới điều tương tự như trường hợp h = r.
Bài 9. Tìm số nguyên dương n sao cho n|3 n - 2 n
Lời giải
Gọi t là một nghịch đảo modulo p của 2 trong đó p là ước nguyên tố nhỏ nhất của n và thỏa
1 ⩽ t ⩽ p - 1 .Khi đó xét 3 n ≡ 2 n (mod p). Ngoài ra 2t ≡ 1(mod p) nên
(2t) n ≡ 2 n t n ≡ 3 n t n ≡ 1(mod p)
Hay (3t) n ≡ 1(mod p). Đặt h = ord 3t p thì h|n. Ngoài ra ta xét nếu 3|n thì vô lý vì 2 n ⋮ 3.
Dẫn tới (3, p) = 1 nên (3t, p) = 1 . Từ đây áp dụng định lý Fermat bé thì
(3t) p-1 ≡ 1(mod p)
Xét h|(p - 1, n) nhưng n > p - 1 nên nếu h > 1 thì tồn tại 1 ước khác của p - 1 và cũng là
ước của n nhưng điều này mâu thuẫn với tính nhỏ nhất của p nên ta có h = 1. Đồng nghĩa
với 3t ≡ 1(mod p). Vô lý với tính duy nhất của t vì 3, 2 là 2 đại diện cho 2 lớp modulo khác
nhau nên nghịch đảo của chúng là duy nhất theo modulo p. Vậy t không tồn tại và n cũng
không có ước nguyên tố tức là n = 1.
Bài 10. Tìm n ∈ N sao cho n > 1 và 3 n - 1 ⋮ n 3 .
Lời giải.
Ta sẽ chứng minh n là số chẵn. Trước hết thì ta gọi p là ước nguyên tố nhỏ nhất của n. Khi
đó 3 n ≡ 1(mod p). Ngoài ra (n, 3) = 1 nên (p, 3) = 1 dẫn tới 3 p-1 ≡ 1(mod p) nên
3 (p-1,n) ≡ 3 ≡ 1(mod p) . Dẫn tới p = 2 và n chẵn. Áp dụng bổ đề LTE, ta có 3 - 1 ⋮ 2 và
đều là các số lẻ nên
v2 3 n - 1 = v2 (3 - 1) + v2 (3 + 1) + v2 (n) - 1 = v2 (n) + 2 ⩾ 3v2 (n)
Nên v 2 (n) = 1. Khi đó đặt n = 2m. Gọi q là một ước nguyên tố khác của m. Xét 3 2m - 1 ⋮ m
hay 3 m - 1 ⋮ m . Tức là
3 m ≡ 1(mod q)
Nhưng do (3, q) = 1 nên 3 q-1 ≡ 1(mod q) dẫn tới 3 (q-1,m) ≡ 3 ≡ 1(mod q) tức q = 2 → vô
lý vì m lẻ. Vậy m = 1 hay n = 2 là số duy nhất thỏa.

You might also like